文章目錄
  1. 1. 数学归纳法
  2. 2. 有理数集的分割
  3. 3. 确界的定义与性质
  4. 4. 含有绝对值的不等式
  5. 5. 绝对误差和相对误差

数学分析习题集(根据2010年俄文版翻译) 在线阅读 Б.П.吉米多维奇 高等教育出版社

$1$.数学归纳法. 为了证明某定理对任意的自然数$n$为真,只需证明下面两点即可:($1$)此定理对$n=1$为真,($2$)若此定理对任何的一个自然数$n$为真,则它对下一个正整数$n+1$也为真.

$2$.分割. 若分有理数为$A$和$B$两类,使其满足于下列条件:($1$)两类均非空集;($2$)每一个有理数必属于一类,且仅属于一类;($3$)属于$A$类(下类)的任一数小于属于$B$类(上类)的任意一个数,则这样的一个分类法称为分割.分割$A/B$确定一个数:($a$)若或是下类$A$有最大的数,或是上类$B$有最小的数,则该分割确定一个有理数;($b$)若$A$类无最大数,而$B$类亦无最小数,则该分割确定一个无理数.有理数和无理数统称为实数.

$3$.绝对值(或模). 若$x$为实数,则下列条件所确定的非负数$\vert x\vert $称为$x$的绝对值(模):

$$\vert x\vert =\begin{cases} -x, & x < 0, \\ x, & x \geq 0. \end{cases} $$

对于任何的实数$x$和$y$,以下不等式成立:

$$\vert x\vert -\vert y\vert \leq \vert x+y\vert \leq \vert x\vert +\vert y\vert .$$

$4$上确界和下确界. 设$X=\lbrace x\rbrace $为实数的有界集合.若:

($1$)每一个$x\in X$满足不等式

$$x\geq m;$$

($2$)对于任何的$\varepsilon > 0$,存在有$x’\in X$,使

$$x’ < m+\varepsilon ,$$

则数

$$m=\inf \lbrace x\rbrace $$

称为集合$X$的下确界.

类似地,若:

($1$)每一个$x\in X$满足不等式

$$x\leq M;$$

($2$)对于任何的$\varepsilon > 0$,存在有$x’’\in X$使

$$x’’ > M-\varepsilon ,$$

则数

$$M=\sup \lbrace x\rbrace $$

称为集合$X$的上确界.

若集合$X$下方无界,则通常说

$$\inf \lbrace x\rbrace =-\infty ;$$

若集合$X$上方无界,则认为

$$\sup \lbrace x\rbrace =+\infty .$$

$5$.绝对误差和相对误差. 设$a(a\neq 0)$是被测量的准确数值,而$x$是这个量的近似值,则

$$\Delta =\vert x-a\vert $$

称为被测量的绝对误差,而

$$\delta =\dfrac{\Delta }{\vert a\vert }$$

称为相对误差.

若$x$的绝对误差不超过它的第$n$个有效数字所对应的位数的单位的一半,则说$x$有$n$位精确数字.

数学归纳法

利用数学归纳法证明下列等式对任何正整数$n$皆成立:

$1$.$1+2+\cdots +n=\dfrac{n(n+1)}{2} $.

证:当$n=1$时,等式成立.

设对于$n=k$(正整数)时,等式成立,即

$$1+2+\cdots +k=\dfrac{k(k+1)}{2} ,$$

则对于$n=k+1$时,有

$$1+2+\cdots +k+(k+1)=\dfrac{k(k+1)}{2} +k+1 =\dfrac{(k+1)[(k+1)+1]}{2} .$$

即对于$n=k+1$时等式也成立.

于是,由数学归纳法知,对于任何正整数$n$,有$1+2+\cdots +n =\dfrac{n(n+1)}{2} $.

$2$.$1^2+2^2+\cdots +n^2=\dfrac{n(n+1)(2n+1)}{6} $.

证:当$n=1$时,等式成立.

设$n=k$时,等式成立,即$1^2+2^2+\cdots +k^2=\dfrac{k(k+1)(2k+1)}{6} $,则对于$n=k+1$时,有

$$\begin{align}
& 1^2+2^2+\cdots +k^2+(k+1)^2 \\
= & \dfrac{k(k+1)(2k+1)}{6} +(k+1)^2 \\
= & \dfrac16 (k+1)[k(2k+1)+6(k+1)] \\
= & \dfrac{(k+1)(2k^2+7k+6)}{6} \\
= & \dfrac{(k+1)(k+2)(2k+3)}{6} \\
= & \dfrac{(k+1)[(k+1)+1] [2(k+1)+1]}{6} ,
\end{align}$$

即对于$n=k+1$时,等式也成立.

于是,对于任何正整数$n$,有$1^2+2^2+\cdots +n^2=\dfrac{n(n+1)(2n+1)}{6} $.

$3$.$1^3+2^3+\cdots +n^3 =(1+2+\cdots +n)^2 $.

证:当$n=1$时,等式成立.

设$n=k$时,等式成立,即$1^3+2^3+\cdots +k^3 =(1+2+\cdots +k)^2 $,则对于$n=k+1$时,有

$$\begin{align}
& 1^3+2^3+\cdots +k^3 +(k+1)^3 \\
= & (1+2+\cdots +k)^2+(k+1)^3 \\
= & \dfrac{k^2(k+1)^2}{4} +(k+1)^3 \\
= & \dfrac{(k+1)^2(k+2)^2}{4} \\
= & \left\lbrace \dfrac{(k+1) [(k+1)+1]}{2} \right\rbrace ^2 \\
= & [1+2+\cdots +(k+1)]^2 ,\\
\end{align} $$

即对于$n=k+1$时,等式也成立.

于是,对于任何正整数$n$,有$1^3+2^3+\cdots +n^3 =(1+2+\cdots +n)^2 $.

$4$.$1+2+2^2+\cdots +2^{n-1} =2^n -1$.

证:当$n=1$时,等式成立.

设$n=k$时,等式成立,即$1+2+2^2+\cdots +2^{k-1} =2^k -1$,则对于$n=k+1$时,有

$$1+2+2^2+\cdots +2^{k-1}+2^k =(2^k-1)+2^k=2^{k+1} -1,$$

即对于$n=k+1$时,等式也成立.

于是,对于任何正整数$n$,有$1+2+2^2+\cdots +2^{n-1} =2^n-1$.

$5$.设$a^{[n]} =a(a-h)\cdots [a-(n-1)h]$及$a^{[0]} =1$,求证

$$\displaystyle (a+b)^{[n]} =\sum_{m=0}^n C_n^m a^{[n-m]} b^{[m]} ,$$

其中$C_n^m$是由$n$个元素中选取$m$个的组合数,由此推出牛顿的二项式公式.

证:当$n=1$时,由于$(a+b)^{[1]} =a+b $及$\displaystyle =\sum_{m=0}^1 C_1^m a^{[1-m]} b^{[m]} =a+b$,所以等式成立.

设$n=k$时,等式成立,即

$$\displaystyle (a+b)^{[k]} =\sum_{m=0}^k C_k^m a^{[k-m]} b^{[m]},\label{1} \tag{1}$$

则对于$n=k+1$时,有

$$(a+b)^{[k+1]} =(a+b)^{[k]} \cdot (a+b-kh).\label{2} \tag{2}$$

将$\eqref{1}$式代入$\eqref{2}$式得

$$\begin{align}
& (a+b)^{[k+1]} \\
= & (a+b-kh)\cdot \displaystyle \sum_{m=0}^k C_k^m a^{[k-m]} b^{[m]} \\
= & (a+b-kh)\left\lbrace C_k^0 a^{[k]} b^{[0]} +C_k^1 a^{[k-1]} b^{[1]} +\cdots +C_k^k a^{[0]} b^{[k]} \right\rbrace \\
= & \left\lbrace (a-kh)+b \right\rbrace C_k^0 a^{[k]} b^{[0]} +\left\lbrace [a-(k-1)h]+(b-h)\right\rbrace C_k^1 a^{[k-1]} b^{[1]} +\cdots + \left\lbrace a+(b-kh)\right\rbrace C_k^k a^{[0]} b^{[k]} \\
= & C_k^0 a^{[k+1]} b^{[0]}+C_k^0 a^{[k]} b^{[1]} +C_k^1 a^{[k]} b^{[1]} +C_k^1 a^{[k-1]} b^{[2]} +\cdots +C_k^k a^{[1]} b^{[k]} +C_k^k a^{[0]} b^{[k+1]} \\
= & C_{k+1}^0 a^{[k+1]} b^{[0]} +(C_k^0 +C_k^1 )a^{[k]} b^{[1]}+\cdots +(C_k^{k-1} +C_k^k )a^{[1]} b^{[k]} +C_{k+1}^{k+1} a^{[0]} b^{[k+1]} \\
& \left( \because C_n^m =C_n^{n-m} =C_{n-1}^{m-1} +C_{n-1}^m \right) \\
= & C_{k+1}^0 a^{[k+1]} b^{[0]} +C_{[k+1]}^1 a^{[k]} b^{[1]} +\cdots +C_{k+1}^k a^{[1]} b^{[k]} +C_{k+1}^{k+1} a^{[0]} b^{[k+1]} \\
= & \sum_{m=0}^{k+1} C_{k+1}^m a^{[k+1-m]} b^{[m]} ,
\end{align}$$

故由$(a+b)^{[k]} =\displaystyle \sum_{m=0}^k C_k^m a^{[k-m]} b^{[m]} $可推得下式成立:

$$(a+b)^{[k+1]} =\displaystyle \sum_{m=0}^{k+1} C_{k+1}^m a^{[k+1-m]} b^{[m]} ,$$

即对于$n=k+1$时,等式也成立.

于是,对于任何正整数$n$,有

$$(a+b)^{[n]} =\displaystyle \sum_{m=0}^n C_n^m a^{[n-m]} b^{[m]} .\label{3} \tag{3} $$

在式子$a^{[n]} =a(a-h)\cdots [a-(n-1)h]$中,令$h=0$,即得

$$a^{[n]} =a^n .\label{4} \tag{4} $$

将$\eqref{4}$式代入$\eqref{3}$式,得牛顿二项式公式$(a+b)^n =\displaystyle \sum_{m=0}^n C_n^m a^{n-m} b^m $.

$6$.证明贝努里不等式

$$(1+x_1 )(1+x_2 )\cdots (1+x_n ) \geq 1+x_1 +x_2 +\cdots +x_n ,$$

式中$x_1 ,x_2 ,\cdots ,x_n $是符号相同且大于$-1$的数.

证:当$n=1$时,此式取等号.

设$n=k$时,不等式成立,即

$$(1+x_1 )(1+x_2 )\cdots (1+x_k ) \geq 1+x_1 +x_2 +\cdots +x_k ,$$

则对于$n=k+1$时,由于$x_i (i=1,2,\cdots ,n)$大于$-1$,所以$1+x_i > 0$.因而有

$$\begin{align}
& (1+x_1 )(1+x_2 )\cdots (1+x_k )(1+x_{k+1} )\\
\geq & (1+x_1 +x_2 +\cdots +x_k )(1+x_{k+1} )\\
= & (1+x_1 +x_2 +\cdots +x_k )+(x_{k+1} +x_1 x_{k+1} +\cdots +x_k x_{k+1} ) .\\
\end{align}$$

由于$x_i x_j \geq 0$,所以

$$(1+x_1 )(1+x_2 )\cdots (1+x_{k+1} )\geq 1+x_1 +x_2 +\cdots +x_{k+1} ,$$

即对于$n=k+1$时,不等式也成立,

于是,对于任何自然数$n$,有

$$(1+x_1 )(1+x_2 )\cdots (1+x_n )\geq 1+x_1 +x_2 +\cdots +x_n .$$

$7$.证明若$x > -1$,则不等式

$$(1+x)^n \geq 1+nx \quad (n > 1)$$

为真,且仅当$x=0$时,等号成立.

证:对于$n=2$,有

$$(1+x)^2=1+2x+x^2 \geq 1+2x,$$

当且仅当$x=0$时等号成立.

现设$n=k$时不等式成立,则当$n=k+1$时有

$$\begin{align}
(1+x)^{k+1} = & (1+x)^k (1+x) \\
\geq & (1+kx)(1+x)=1+(k+1)x+kx^2 \\
\geq & 1+(k+1)x,
\end{align} $$

同样从归纳假设和上式推导可知当且仅当$x=0$时等号成立.

这里有一个意外,当$-2\leq x\leq -1$时本题的伯努利不等式仍然成立.

证:令$\psi _n (x)=(1+x)^n -1-nx$,其中$x\leq -1$.

如果$n$为偶数,因为$x\leq -1$,所以$-nx\geq 1$.这时,

$$\psi _n (x)=(1+x)^n -1-nx\geq (1+x)^n-1+1=(1+x)^n \geq 0.$$

如果$n$为奇数,对$\psi _n (x)$求关于$x$的一阶导,并令

$$\psi’ _n (x) =n(1+x)^{n-1} =n[(1+x)^{n-1} -1]=0,$$

可解出$x=-2$.可知,

当$x\in (-\infty ,-2)$,$\psi’ _n (x) > 0$,$\psi _n (x)$单调递增$\nearrow$;

当$x\in (-2 ,-1)$,$\psi’ _n (x) < 0$,$\psi _n (x)$单调递减$\searrow$;

因此,当$x\in (-2 ,-1)$时,有

$$\psi _n (x) > \psi _n (-1) =[1+(-1)]^n-1-(-1)n\geq 0.$$

特别地,当$x=-1$时,$\psi _n (-1)=n-1 \geq 0$;

当$x=-2$时,

$$\psi _n (-2) =[1+(-2)]^n -1-(-2)n=(-1)^n-1+2n \geq (-1)^n +1 \geq 0.$$

综上所述,当$x\in [-2,-1]$时,都有$\psi _n (x)\geq 0$成立,即对于任意的自然数$n$,有

$$(1+x)^n\geq 1+nx.$$

$8$.证明不等式:

$$n! < \left( \dfrac{n+1}{2} \right) ^n \quad (n > 1).$$

提示:利用不等式

$$\left( \dfrac{n+2}{n+1} \right) ^{n+1} =\left( 1+\dfrac{1}{n+1} \right) ^{n+1} > 2\quad (n=1,2,\cdots ).$$

证:当$n=2$时,因为$\left( \dfrac{2+1}{2} \right) ^2 =\dfrac94 > 2=2!$,故不等式成立.

设$n=k$时,不等式成立,则

$$k! < \left( \dfrac{k+1}{2} \right) ^k ,$$

则对于$n=k+1$时,有

$$(k+1)! < \left( \dfrac{k+1}{2} \right) ^k (k+1) =2\left( \dfrac{k+1}{2} \right) ^{k+1} .$$

由于

$$\left( \dfrac{k+2}{k+1} \right) ^{k+1} =\left( 1+\dfrac{1}{k+1} \right) ^{k+1} > 2(k=1,2,\cdots ),$$

从而有

$$(k+1)! < \left[ \dfrac{(k+1)+1}{2} \right] ^{k+1} ,$$

即对于$n=k+1$时,不等式也成立.

于是,对于任何自然数$n$,有

$$n! < \left( \dfrac{n+1}{2} \right) ^n .$$

$9$.证明如下不等式:

$(a)\;2!\cdot 4!\cdots (2n)! > [(n+1)!]^n \;(n > 1)$;

$(b)\;\dfrac12 \cdot \dfrac34 \cdots \dfrac{2n-1}{2n} < \dfrac{1}{\sqrt{2n+1} } $.

证:$(a)\;$当$n=2$时,因为$2!\cdot 4! =48$,及$[(2+1)!]^2 =36$,所以$2!\cdot 4! > [(2+1)!]^2$,故不等式成立.

设$n=k$时,不等式成立,即

$$2!\cdot 4!\cdots (2k)! > [(k+1)!]^k,$$

则对于$n=k+1$时,有

$$\begin{align}
& 2!\cdot 4!\cdots (2k+2)! \\
\,> & [(k+1)!]^k (2k+2)! \\
= & [(k+1)!]^{k+1} (k+2)(k+3)\cdots (2k+2) \\
\,> & [(k+1)!]^{k+1} (k+2)^{k+1} \\
= & [(k+2)!]^{k+1} ,
\end{align} $$

即对于$n=k+1$时,不等式也成立.于是,由数学归纳法原理,本题证毕.

$(b)\;$证$1\quad $当$n=1$时,因为$\dfrac12 < \dfrac{1}{\sqrt{3} }$,不等式显然成立.

设$n=k$时,不等式成立,即$\dfrac12 \cdot \dfrac34 \cdots \dfrac{2k-1}{2k} < \dfrac{1}{\sqrt{2k+1} } $.

对于$n=k+1$而言,由于

$$\dfrac12 \cdot \dfrac34 \cdots \dfrac{2k+1}{2k+2} < \dfrac{1}{\sqrt{2k+1} } \cdot \dfrac{2k+1}{2k+2} =\dfrac{\sqrt{2k+1} }{2k+2} .$$

故只要证$\dfrac{\sqrt{2k+1} }{2k+2} < \dfrac{1}{\sqrt{2k+3} } $,即证$(2k+1)(2k+3) < (2k+2)^2 $,而上述不等式由于

$$4k^2 +8k+3 < 4k^2 +8k+4,$$

因而是成立的.于是,最后得

$$\dfrac12 \cdot \dfrac34 \cdots \dfrac{2k+1}{2k+2} < \dfrac{1}{\sqrt{2k+3} } ,$$

即对于$n=k+1$时,不等式也成立.于是,由数学归纳法,本题证毕.

证$2\quad $利用不等式

$$\dfrac{\sqrt{(2k-1)(2k+1)}}{2k} < 1,$$

令$k=1,2,\cdots ,n$,并将所得的不等式边相乘得到

$$\dfrac{\sqrt{1\cdot 3}}{2} \cdot \dfrac{\sqrt{3\cdot 5}}{4} \cdot \cdots \cdot \dfrac{\sqrt{(2n-1)(2n+1)}}{2n} =\dfrac12 \cdot \dfrac34 \cdot \cdots \cdot \dfrac{2n-1}{2n} \cdot \sqrt{2n+1} < 1,$$

整理后即得.

证$3\quad $记$x_n =\dfrac{1\cdot 3\cdot \cdots \cdot (2n-1)}{2\cdot 4\cdot \cdots \cdot (2n)}$.利用$\dfrac{2k-1}{2k} < \dfrac{2k}{2k+1}$,$k=1,2,\cdots ,n$,就有

$$x_n^2 < x_n \cdot \dfrac{2\cdot 4\cdot \cdots \cdot (2n)}{3\cdot 5\cdot \cdots \cdot (2n+1)} =\dfrac{1}{2n+1} ,$$

两边开平方即得.

$10$.证明如下不等式:

$(a)\;1+\dfrac{1}{\sqrt{2}} +\dfrac{1}{\sqrt{3}} +\cdots +\dfrac{1}{\sqrt{n}} > \sqrt{n} \;(n\geq 2)$;

$(b)\;n^{n+1} > (n+1)^n \;(n\geq 3)$;

$(c)\;$ $\displaystyle \Bigg| \sin{\left( \sum_{k=1}^n x_k \right) } \Bigg| \leq \sum_{k=1}^n \sin{x_k } \;(0\leq x_k \leq \pi ,k=1,2,\cdots ,n)$;

$(d)\;(2n)! < 2^{2n} (n!)^2$.

证:$(a)\;$证$1\quad $对于$n=2$可以从

$$1+\dfrac{1}{\sqrt{2}} > \sqrt{2} \iff \sqrt{2} +1 > 2$$

直接看出成立.

现设$n=k$时成立,则对$n=k+1$就有

$$\begin{align}
& 1+\dfrac{1}{\sqrt{2}} +\cdots +\dfrac{1}{\sqrt{k}} +\dfrac{1}{\sqrt{k+1}} \\
\,> & \sqrt{k} +\dfrac{1}{\sqrt{k+1}} \\
= & \dfrac{1}{\sqrt{k+1}} \cdot (\sqrt{k(k+1)} +1) \\
\,> & \dfrac{1}{\sqrt{k+1}} \cdot (\sqrt{k^2} +1) \\
= & \sqrt{k+1} .
\end{align} $$

证$2\quad $在$n\geq 2$时将每一项换为$\dfrac{1}{\sqrt{n}}$即得所求:

$$1+\dfrac{1}{\sqrt{2}}+\dfrac{1}{\sqrt{3}}+\cdots +\dfrac{1}{\sqrt{n}} > \dfrac{1}{\sqrt{n}}+\dfrac{1}{\sqrt{n}}+\cdots +\dfrac{1}{\sqrt{n}} =\dfrac{n}{\sqrt{n}} =\sqrt{n} .$$

证$3\quad $利用裂项消去法.从

$$\dfrac{1}{\sqrt{k}} =\dfrac{2}{2\sqrt{k}} > \dfrac{2}{\sqrt{k} +\sqrt{k+1} } =2(\sqrt{k+1} -\sqrt{k} )$$

出发,令其中的$k=1,2,\cdots ,n$,将所得的不等式边边相加得到

$$1+\dfrac{1}{\sqrt{2}}+\cdots +\dfrac{1}{\sqrt{n}} > 2(\sqrt{n+1} -1).$$

对于$n=2$上式右边已大于$\sqrt{2}$,对于$n\geq 3$则可以从

$$2(\sqrt{n+1} -1)=\sqrt{n+1} +(\sqrt{n+1} -2) \geq \sqrt{n+1} > \sqrt{n}$$

得到.

$(b)\;$证$1\quad $对$n=3$即是$3^4 =81 > 4^3 =64$(对$n=2$不等式不成立).

现设$n=k$时不等式成立,于是我们需要证明的是$n=k+1$时成立不等式

$$(k+1)^{k+2} > (k+2)^{k+1} .\label{5} \tag{5} $$

为了利用归纳假设$k^{k+1} > (k+1)^k$,将$\eqref{5}$的左边乘除$k^{k+1}$,即可推导如下:

$$\begin{align}
(k+1)^{k+2} = & \dfrac{(k+1)^{k+2} }{k^{k+1}} \cdot k^{k+1}\\
\,> & \dfrac{(k+1)^{k+2} }{k^{k+1}} \cdot (k+1)^k = \left( \dfrac{(k+1)^2}{k} \right) ^{k+1} \\
\,> & \left( \dfrac{k^2+2k}{k} \right) ^{k+1} = (k+2)^{k+1} .\\
\end{align} $$

证$2\quad $首先将不等式改写为等价的$\left( 1+\dfrac{1}{n} \right) ^n < n$,然后利用已知数列$\left\lbrace \left( 1+\dfrac{1}{n} \right) ^n \right\rbrace $是单调递增数列,且收敛于$e\approx 2.718\;28 < 3$,可见,当$n \geq 3$时不等式成立.

$(c)\;$对于$n=1$由于$0\leq x_1 \leq \pi $,因此$\sin{x_1 } \geq 0$,不等式以等号成立.

现设$n$时不等式成立,则当$n+1$时就有

$$\begin{align}
\displaystyle \Bigg| \sin{\left( \sum_{k=1}^{n+1} x_k \right) } \Bigg| = & \Bigg| \sin{\left( \sum_{k=1}^n x_k +x_{n+1} \right) } \Bigg| \\
= & \Bigg| \sin{\left( \sum_{k=1}^n x_k \right) } \cos{x_{n+1} } +\cos{\left( \sum_{k=1}^n x_k \right) } \sin{x_{n+1}} \Bigg| \\
\leq & \Bigg| \sin{\left( \sum_{k=1}^n x_k \right) } \cos{x_{n+1} } \Bigg| +\Bigg| \cos{\left( \sum_{k=1}^n x_k \right) } \sin{x_{n+1}} \Bigg| \\
\leq & \Bigg| \sin{\left( \sum_{k=1}^n x_k \right) } \Bigg| +\Bigg| \sin{x_{n+1}} \Bigg| \leq \sum_{k=1}^{n+1} \sin{x_k } .
\end{align} $$

这里最后一步利用了$n=k$时的归纳假设,还利用了$0\leq x_{n+1} \leq \pi $时$\sin{x_{n+1}} \geq 0$.

$(d)\;$证$1\quad $对$n=1$不等式就是$2 < 4$.

现设$n=k$时不等式成立,则对$n=k+1$就有

$$\begin{align}
(2(k+1))! = & (2k)!\cdot (2k+1)(2k+2) \\
< & 2^{2k}(k!)^2\cdot (2k+1)(2k+2) \\
< & 2^{2k}(k!)^2\cdot (2k+2)^2 \\
= & 2^{2(k+1)}[(k+1)!]^2 .
\end{align} $$

证$2\quad $将左边作如下分解即可证出所要的不等式

$$(2n)!=(2n)!!(2n-1)!! < [(2n)!!]^2 =2^{2n} (n!)^2.$$

科普读物:华罗庚.数学归纳法.//华罗庚.华罗庚科普著作选集.上海:上海教育出版社,1984
Соминскии И С.Математической Индукции.Москва:ТЕХГИЗ,1952(中译本:索明斯基.数学归纳法.北京:中国青年出版社,1953)

有理数集的分割

$11$.设$c$为正整数,而不为整数的平方,且$A/B$为确定实数$\sqrt{c}$的分割,其中$B$类包含所有满足$b^2 > c$的正有理数$b$,而$A$类包含所有其余的有理数.求证:在$A$类中无最大数,而在$B$类中无最小数.

证明:设$a\in A$.若$a\leq 0$,则显然存在$a’ > a(a’ > 0)$且$a’ \in A$.故可设$a > 0$,于是$a^2 \leq c$.但不可能有$a^2 =c$.因若$a^2=c$,设$a=\dfrac{p}{q} $,$p$与$q$为互素的正整数,则$\dfrac{p^2}{q^2} =c$.由于$c$是正整数,而$p^2$与$q^2$也是互素的,故必$q^2=1$,从而$c=p^2$,此与假定矛盾,故必$a^2 < c$.下面我们证明,存在正整数$n$,使

$$\left( a+\dfrac{1}{n} \right) ^2 < c,$$

于是$a+\dfrac{1}{n}$也属于$A$.

上述不等式相当于:$a^2 +\dfrac{2a}{n} +\dfrac{1}{n^2} < c$,$\dfrac{2a}{n} +\dfrac{1}{n^2} < c-a^2$,若$n$满足不等式$\dfrac{2a+1}{n} < c-a^2$,则上面的第二个不等式也自然能满足了.

$$a^2 +\dfrac{2a}{n} +\dfrac{1}{n^2} < a^2 +\dfrac{2a}{n} +\dfrac{1}{n} < c$$

为此,只要取$n > \dfrac{2a+1}{c-a^2} $,而这是恒为可能的.因此,不论$a$为$A$类内怎样的数,在$A$类内总能找到大于它的数,故$A$类中无最大数.

同法可证$B$类中无最小数.

设正整数$b\in B$.下面我们证明,存在正整数$m$,使

$$\left( b-\dfrac{1}{m} \right) ^2 > c,$$

于是$b-\dfrac{1}{m}$也属于$B$.

上述不等式相当于:$b^2 -\dfrac{2b}{m} +\dfrac{1}{m^2} > c$,$-\dfrac{2b}{m} +\dfrac{1}{m^2} > c-b^2$,若$m$满足不等式$-\dfrac{2b}{m} > c-b^2$,则上面的第二个不等式也自然能满足了.

为此,只要取$m > \dfrac{2b}{b^2-c} $,就足够了.因此,不论$b$为$B$类内怎样的数,在$B$类只要取

$$m > \max{\left\lbrace \dfrac{1}{b} ,\dfrac{2b}{b^2-c} \right\rbrace } $$

就足以保证有$b-\dfrac{1}{m} \in B$.故$B$类中无最小数.

实质上,此处分割$A/B$确定了一个无理数$\sqrt{c}$.

$12$.确定数$\sqrt[3]{2}$的分割$A/B$用下面的方法建立:$A$类包含所有满足$a^3 < 2$的有理数$a$,$B$类包含所有其余的有理数.证明:在$A$类中无最大数,而在$B$类无最小数.

证明:设$a\in A$,即$a^3 < 2$.下证必可取正整数$n$,使

$$\left( a+\dfrac{1}{n} \right) ^3 < 2.$$

事实上,上式相当于$\dfrac{3a^2}{n} +\dfrac{3a}{n^2} +\dfrac{1}{n^3} < 2-a^3$.若$a\leq 0$,取$n=1$即可.若$a > 0$,注意到$n \leq 1$,即知若取$n$充分大,使$n > \dfrac{3a^2 +3a+1}{2-a^3} $,则上列各式均成立.从而$a+\dfrac{1}{n} \in A$.故$A$中无最大数.

下证$b\in B$,则$b^3 \geq 2$.下证不可能有$b^3 =2$.事实上,若$b^3 =2$,设$b=\dfrac{p}{q}$,$p$与$q$为互素的正整数,则$\dfrac{p^3}{q^3} =2$,$p^3 =2q^3$,从而$p^3$为偶数,因此$p$必为偶数:$p=2r$,$r$为正整数.由于$p$与$q$是互素的,故$q$必为奇数,从而$q^3$也为奇数,但$q^3 =4r^3$,故$q^3$又必是偶数,因此矛盾.由此可知必有$b^3 > 2$.仿前面的证明,可取正整数$n$,使$\left( b-\dfrac{1}{n} \right) ^3 > 2$,从而$b-\dfrac{1}{n} \in B$.由此可知$B$类中无最小数.实质上,此处分割$A/B$确定了一个无理数$\sqrt[3]{2}$.

$13$.作出适当的分割,然后证明等式:

$(a)\;\sqrt{2} +\sqrt{8} =\sqrt{18} $;$(b)\;\sqrt{2} \sqrt{3} =\sqrt{6} $.

证明:$(a)\;$作确定$\sqrt{2}$的分割$A/B$:一切有理数$a\leq 0$以及满足$a^2 < 2$的正有理数$a$都归于$A$类,一切满足$b^2 > 2$的正有理数$b$归入$B$类.又作确定$\sqrt{8}$的分割$A’/B’$:一切有理数$a’ \leq 0$以及满足${a’}^2 < 8$的正有理数$a’$归入$A’$类,一切满足${b’}^2 > 8$的正有理数$b’$归入$B’$类.我们知道,根据实数加法的定义,满足不等式:

$$a+a’ < c < b+b’(\forall a \in A ,b\in B ,a’\in A’ ,b’\in B’)$$

的唯一实数$c$就是$\sqrt{2} +\sqrt{8}$.因此,如果我们能证明恒有$(a+a’)^2 < 18$(当$a+a’ > 0$时),$(b+b’)^2 > 18$,则有$a+a’ < \sqrt{18} < b+b’$.于是得知$\sqrt{18} =c=\sqrt{2} +\sqrt{8} $.

若$a+a’ > 0$,则$a$与$a’$中至少有一个为正,从而由$a^2{a’}^2 < 16$知$aa’ < 4$,

$$(a+a’)^2 =a^2 +{a’}^2 +2aa’ < 2+8+8 =18;$$

同样,因$b^2 > 2$,${b’}^2 > 8$,$b > 0$,$b’ > 0$,故$b^2{b’}^2 > 16 $,$bb’ > 4$,

$$(b+b’)^2=b^2+{b’}^2+2bb’ > 2+8+8 =18.$$

于是证毕.

$(b)\;$作确定$\sqrt{2}$的分割$A/B$如$(a)$中所示,再作确定$\sqrt{3}$的分割$A_1 /B_1 $:一切有理数$a_1 \leq 0$以及满足$a_1^2 < 3$的正有理数$a_1 $归入$A_1 $类,一切满足$b_1^2 > 3$的正有理数$b_1 $归入$B_1 $类,根据实数乘法的定义,满足

$$aa_1 < c_1 < bb_1 (\forall a\in A ,a > 0 ,a_1 \in A_1 ,a_1 > 0 ,b\in B ,b_1 \in B_1 )$$

的(正)实数$c_1$存在唯一,它就是$\sqrt{2} \cdot \sqrt{3}$.但由于当$a\in A ,a > 0,a_1 \in A_1 ,a_1 > 0$时$(aa_1 )^2 < 6$,而当$b\in B ,b_1 \in B_1 $时,$(bb_1 )^2 > 6$,故恒有$aa_1 < \sqrt{6} < bb_1 $.由此可知$\sqrt{6} =c_1 =\sqrt{2} \cdot \sqrt{3} $.证毕.

$14$.建立确定数$2^{\sqrt{2}}$的分割.

解:设$A’/B’$是确定$\sqrt{2}$的有理数系中的一个分割.定义如下的两个有理数集:

$$A=\lbrace a\in \mathbb{Q} \mid a\leq 0 或存在 \dfrac{p}{q} \in A’,p,q都是正整数,使得a^q < 2^p \rbrace ,$$

$$B=\lbrace b\in \mathbb{Q} \mid b > 0 ,存在 \dfrac{p’}{q’} \in B’,p’,q’都是正整数,使得b^{q’} < 2^{p’} \rbrace ,$$

则当$a\in A$和$b\in B$时,由于$A,B$的定义可知有$a < 2^{\frac{p}{q}} $和$2^{\frac{p’}{q’}} < b$,而从分割$A’/B’$知道有$\dfrac{p}{q} < \dfrac{p’}{q’} $,因此$a < b$.

如果我们再能够证明$A\cup B =\mathbb{Q}$,则$A/B$就是有理数系中的一个戴德金分割.用反证法.

设存在一个有理数$x$,它既不属于$A$,也不属于$B$,于是对于任何$\dfrac{p}{q} \in A’$和$\dfrac{p’}{q’} \in B’$,就有

$$2^{\frac{p}{q}} < x < 2^{\frac{p’}{q’}} .$$

由于$A’$中没有最大数,$B’$中没有最小数,因此上述两个不等号都只能是严格的$<$号.

于是如分析中所说,这个有理数$x$只能是数$2^{\sqrt{2}}$本身.根据希尔伯特第七问题的答案,这样的有理数$x$是不存在的.于是就证明了上述$A/B$确实是定义$2^{\sqrt{2}}$的戴德金分割.

若不考虑希尔伯特第七问题的结论,则可以解答如下.

情况一.如果$2^{\sqrt{2}}$不是有理数,则上面的推理已经证明了$A/B$就是确定它的分割.

情况二.如果$2^{\sqrt{2}}$是有理数,则将它加入$A$或$B$之后,也就得到确定这个有理数的两个可能的分割.

确界的定义与性质

$15$.求证:任何非空且下方有界的数集有下确界,而任何非空且上方有界的数集有上确界.

证:$(1)\;$设$X$是有下界的数集,于是集合

$$A=\lbrace x\in \mathbb{R} \mid x是X的一个上界\rbrace $$

非空,且以$X$中的任何一个数为其上界.然后定义

$$B=\lbrace x\in \mathbb{R} \mid x\notin A\rbrace .$$

不难看出这样定义的$A,B$满足戴德金分割中的前两个条件.下面验证它也满足第三个条件.若$a\in A $,$b\in B$,则由于$b$不是$X$的下界,因此存在某个$x\in X$,使得$x < b$.于是就有$a \leq x < b$成立.

这时的一种可能性是$A$有最大数,也就是$X$有最大下界,即已经证明存在$\inf{X}$.

另一种可能性是$B$有最小数.将它记为$\beta $.这时可以证明每一个$x\in X$都有$x\geq \beta $,否则,若有某个$x_0 \in X$,且$x_0 < \beta $,则$x_0$和$\dfrac{x_0 +\beta }{2}$都只能属于$A$.另一方面从$x_0 < \dfrac{x_0 +\beta }{x} $且$x_0 \in X$可见$\dfrac{x_0 +\beta }{2}$不是$X$的下界,因此与它属于$A$相矛盾.

于是我们已经证明:若$B$有最小数,则这个最小数也是$X$的下界.这表明它也属于$A$.这与$B$是$A$的补集相矛盾.因此这种情况不可能发生.

于是按照上述方式定义的数集$A$一定有最大数,因此$X$一定有最大下界,即$X$存在下确界.

$(2)\;$设$X$是有上界的数集,于是集合

$$B=\lbrace x\in \mathbb{R} \mid x是X的一个上界\rbrace $$

非空,且以$X$中的任何一个数为其下界.然后定义

$$A=\lbrace x\in \mathbb{R} \mid x\notin B\rbrace .$$

不难看出这样定义的$A,B$满足戴德金分割中的前两个条件.下面验证它也满足第三个条件.若$a\in A $,$b\in B$,则由于$a$不是$X$的上界,因此存在某个$x\in X$,使得$a < x$.于是就有$a < x \leq b$成立.

这时的一种可能性是$B$有最小数,也就是$X$有最小上界,即已经证明存在$\sup{X}$.

另一种可能性是$A$有最大数.将它记为$\alpha $.这时可以证明每一个$x\in X$都有$x\leq \alpha $,否则,若有某个$x_0 \in X$,且$x_0 > \alpha $,则$x_0$和$\dfrac{x_0 +\alpha }{2}$都只能属于$B$.另一方面从$\dfrac{x_0 +\alpha }{x} < x_0 \in X$可见$\dfrac{x_0 +\alpha }{2}$不是$X$的上界,因此与它属于$B$相矛盾.

于是我们已经证明:若$A$有最大数,则这个最大数也是$X$的上界.这表明它也属于$B$.这与$A$是$B$的补集相矛盾.因此这种情况不可能发生.

于是按照上述方式定义的数集$B$一定有最小数,因此$X$一定有最小上界,即$X$存在上确界.

$16$.证明:一切有理真分数$\dfrac{m}{n}$(式中$m$及$n$为正整数,且$0 < m < n$)的集合无最小及最大的元素.求此集合的上确界及下确界.

证明:由不等式:

$$\dfrac{1}{n} \leq \dfrac{m}{n} \leq \dfrac{n-1}{n} =1-\dfrac{1}{n} .$$

由此可见只要增大分母,例如取分母为$n+1$,就有$\dfrac{1}{n+1} < \dfrac{m}{n}$,同时又有$\dfrac{m}{n} < \dfrac{n}{n+1}$.因此可知真分数全体的集合中没有最小的数,也没有最大的数.

由于真分数在$0$到$1$之间,对任意给定的$0 < b < c < 1$,只要$n$充分大,就可使$\dfrac{1}{n} < b $,$\dfrac{n-1}{n} =1-\dfrac{1}{n} > c$,因此可知所有真分数集合的下确界为$0$,上确界为$1$.

$17$.求满足不等式

$$r^2 < 2$$

的有理数$r$的下确界和上确界.

解:用$E$表所有满足$r^2 < 2$的有理数$r$所成的集合.我们知道,分割$A/B$确定无理数$\sqrt{2}$,这里$A$表示由一切非正有理数以及满足$r^2 < 2$的正有理数$r$所成的类,$B$表其余有理数构成的类,并且已证$A$中无最大数,于是

$$\sup{E} =\sup{A} =\sqrt{2} .$$

同样,分割$A’/B’$确定无理数$-\sqrt{2}$,这里$B’$表由所有非负有理数以及满足$r^2 < 2$的负有理数$r$构成的类,$A’$表其余有理数构成的类,并且$B’$中无最小数.于是,显然有

$$\inf{E} =\inf{B’} =-\sqrt{2}.$$

$18$.设$\lbrace -x\rbrace $为数$x\in \lbrace x\rbrace $的相反数的集合,证明:

$(a)\;\inf{\lbrace -x\rbrace } =-\sup{\lbrace x\rbrace } ;(b)\;\sup{\lbrace -x\rbrace } =-\inf{\lbrace x\rbrace } $.

证明:$(a)\;$设$\inf{\lbrace -x\rbrace } =m’$,则有:

(Ⅰ)当$-x \in \lbrace -x\rbrace $时,$-x \geq m’$;(Ⅱ)对于任何的正数$\varepsilon $,存在有$-x’\in \lbrace -x\rbrace $,使$-x’ < m’+\varepsilon $.

由(Ⅰ)及(Ⅱ)推得:

(Ⅲ)当$x\in \lbrace x\rbrace $时,$x \leq -m’$;(Ⅳ)对于任何的正数$\varepsilon $,存在有$x’\in \lbrace x\rbrace $,使$x’ > -m’-\varepsilon $.

由(Ⅲ)及(Ⅳ)知数$-m’=\sup{\lbrace x\rbrace } $,即$m’=-\sup{\lbrace x\rbrace } $,所以,$\inf{\lbrace -x\rbrace } =-\sup{\lbrace x\rbrace } $.

$(b)\;$设$\sup{\lbrace -x\rbrace } =M’$,则有:

(Ⅴ)当$-x \in \lbrace -x\rbrace $时,$-x \leq M’$;(Ⅵ)对于任何的正数$\varepsilon $,存在有$-x’\in \lbrace -x\rbrace $,使$-x’ > M’-\varepsilon $.

由(Ⅴ)及(Ⅵ)推得:

(Ⅶ)当$x\in \lbrace x\rbrace $时,$x \geq -M’$;(Ⅷ)对于任何的正数$\varepsilon $,存在有$x’\in \lbrace x\rbrace $,使$x’ < -M’+\varepsilon $.

由(Ⅶ)及(Ⅷ)知数$-M’=\inf{\lbrace x\rbrace } $,即$M’=-\inf{\lbrace x\rbrace } $,所以,$\sup{\lbrace -x\rbrace } =-\inf{\lbrace x\rbrace } $.

$19$.设$\lbrace x+y\rbrace $为所有的和$x+y$的集合,其中$x\in \lbrace x \rbrace $及$y\in \lbrace y\rbrace $,证明等式:

$(a)\;\inf{\lbrace x+y\rbrace } =\inf{\lbrace x\rbrace } +\inf{\lbrace y\rbrace } ;(b)\;\sup{\lbrace x+y\rbrace } =\sup{\lbrace x\rbrace } +\sup{\lbrace y\rbrace }$.

证明:$(a)\;$设$\inf{\lbrace x\rbrace } =m_1 ,\inf{\lbrace y\rbrace } =m_2 $,则有

(Ⅰ)当$x\in \lbrace x\rbrace ,y\in \lbrace y\rbrace $时,$x\geq m_1 ,y\geq m_2 $;

(Ⅱ)对于任何的正数$\varepsilon $,存在有数$x’\in \lbrace x\rbrace ,y’\in \lbrace y\rbrace $,使$x’ < m_1 +\dfrac{\varepsilon }{2} ,y’ < m_2 +\dfrac{\varepsilon }{2}$.

由(Ⅰ)及(Ⅱ)推得:

(Ⅲ)当$x+y \in \lbrace x+y\rbrace $时(其中$x\in \lbrace x\rbrace ,y\in \lbrace y\rbrace $),$x+y \geq m_1 +m_2 $;

(Ⅳ)对于任何的正数$\varepsilon $,存在有数$x’+y’ \in \lbrace x+y\rbrace $(其中$x’\in \lbrace x\rbrace ,y’\in \lbrace y\rbrace $),使$x’+y’ < (m_1 +m_2 )+\varepsilon $.

由(Ⅲ)及(Ⅳ)知数$m_1 +m_2 =\inf{\lbrace x+y\rbrace } $,即$\inf{\lbrace x+y\rbrace } =\inf{\lbrace x\rbrace } +\inf{\lbrace y\rbrace } $.

$(b)\;$设$\sup{\lbrace x\rbrace } =M_1 ,\sup{\lbrace y\rbrace } =M_2 $,则有

(Ⅰ)当$x\in \lbrace x\rbrace ,y\in \lbrace y\rbrace $时,$x\leq M_1 ,y\leq M_2 $;

(Ⅱ)对于任何的正数$\varepsilon $,存在有数$x’\in \lbrace x\rbrace ,y’\in \lbrace y\rbrace $,使$x’ > M_1 -\dfrac{\varepsilon }{2} ,y’ > M_2 -\dfrac{\varepsilon }{2}$.

由(Ⅰ)及(Ⅱ)推得:

(Ⅲ)当$x+y \in \lbrace x+y\rbrace $时(其中$x\in \lbrace x\rbrace ,y\in \lbrace y\rbrace $),$x+y \leq M_1 +M_2 $;

(Ⅳ)对于任何的正数$\varepsilon $,存在有数$x’+y’ \in \lbrace x+y\rbrace $(其中$x’\in \lbrace x\rbrace ,y’\in \lbrace y\rbrace $),使$x’+y’ > (M_1 +M_2 )-\varepsilon $.

由(Ⅲ)及(Ⅳ)知数$M_1 +M_2 =\sup{\lbrace x+y\rbrace } $,即$\sup{\lbrace x+y\rbrace } =\sup{\lbrace x\rbrace } +\sup{\lbrace y\rbrace } $.

$20$.设$\lbrace xy\rbrace $为所有的乘积$xy$的集合,其中$x\in \lbrace x \rbrace $及$y\in \lbrace y\rbrace $,且$x\geq 0$及$y\geq 0$.证明等式:

$(a)\;\inf{\lbrace xy\rbrace } =\inf{\lbrace x\rbrace } \inf{\lbrace y\rbrace } ;(b)\;\sup{\lbrace xy\rbrace } =\sup{\lbrace x\rbrace } \sup{\lbrace y\rbrace }$.

证明:$(a)\;$设$\inf{\lbrace x\rbrace } =m_1 ,\inf{\lbrace y\rbrace } =m_2 $,由于恒有$x \geq 0$,$y\geq 0$,故必$m_1 \geq 0,m_2 \geq 0$.于是,

(Ⅰ)当$x\in \lbrace x\rbrace ,y\in \lbrace y\rbrace $时,$x\geq m_1 \geq 0,y\geq m_2 \geq 0$;

(Ⅱ)对于任何的正数$\varepsilon $,存在有数$x’\in \lbrace x\rbrace ,y’\in \lbrace y\rbrace $,使$0\leq x’ < m_1 +\varepsilon ,0\leq y’ < m_2 +\varepsilon $.

由(Ⅰ)及(Ⅱ)推得:

(Ⅲ)当$xy \in \lbrace xy\rbrace $,其中$x\in \lbrace x\rbrace ,y\in \lbrace y\rbrace $,$xy \geq m_1 m_2 $;

(Ⅳ)对于任何的正数$\varepsilon $,存在有数$x’y’ \in \lbrace xy\rbrace $(其中$x’\in \lbrace x\rbrace ,y’\in \lbrace y\rbrace $),使

$$0\leq x’y’ < (m_1 +\varepsilon )(m_2 +\varepsilon ) =m_1 m_2 +\varepsilon’ ,$$

其中$\varepsilon’ =(m_1 +m_2 )\varepsilon +\varepsilon ^2 $.

由(Ⅲ)及(Ⅳ)知数$m_1 m_2 =\inf{\lbrace xy\rbrace } $,即$\inf{\lbrace xy\rbrace } =\inf{\lbrace x\rbrace } \inf{\lbrace y\rbrace } $.

$(b)\;$设$\sup{\lbrace x\rbrace } =M_1 ,\sup{\lbrace y\rbrace } =M_2 $,由于恒有$x \geq 0$,$y\geq 0$,故必$M_1 \geq 0,M_2 \geq 0$.于是,

(Ⅰ)当$x\in \lbrace x\rbrace ,y\in \lbrace y\rbrace $时,$0\leq x\leq M_1 ,0\leq y\leq M_2 $;

(Ⅱ)对于任何的正数$\varepsilon $,存在有数$x’\in \lbrace x\rbrace ,y’\in \lbrace y\rbrace $,使$x’ > M_1 -\varepsilon \geq 0,y’ > M_2 -\varepsilon \geq 0$.

由(Ⅰ)及(Ⅱ)推得:

(Ⅲ)当$xy \in \lbrace xy\rbrace $,其中$x\in \lbrace x\rbrace ,y\in \lbrace y\rbrace $,$xy \leq M_1 M_2 $;

(Ⅳ)对于任何的正数$\varepsilon $,存在有数$x’y’ \in \lbrace xy\rbrace $(其中$x’\in \lbrace x\rbrace ,y’\in \lbrace y\rbrace $),使

$$x’y’ > (M_1 -\varepsilon )(M_2 -\varepsilon ) =M_1 M_2 +\varepsilon’ \geq 0,$$

其中$\varepsilon’ =(M_1 -M_2 )\varepsilon +\varepsilon ^2 $.

由(Ⅲ)及(Ⅳ)知数$M_1 M_2 =\sup{\lbrace xy\rbrace } $,即$\sup{\lbrace xy\rbrace } =\sup{\lbrace x\rbrace } \sup{\lbrace y\rbrace } $.

含有绝对值的不等式

$21$.求证不等式:

$$(a)\;\vert x-y\vert \geq \big| \vert x\vert -\vert y\vert \big| ;(b)\;\vert x+x_1 +\cdots +x_n \vert \geq \vert x\vert -(\vert x_1 \vert +\cdots +\vert x_n \vert ).$$

证明:$(a)\;$由三点不等式$\vert x+y\vert \leq \vert x\vert +\vert y+\vert $有$\vert x\vert =\vert x-y+y \vert \leq \vert x-y\vert +\vert y\vert $,然后移项得到

$$\vert x-y\vert \geq \vert x\vert -\vert y\vert .$$

交换$x$与$y$后又得到$\vert x-y\vert =\vert y-x\vert \geq \vert y\vert -\vert x\vert $.合并即可.

$(b)\;$用三点不等式$\vert x+y\vert \leq \vert x\vert +\vert y+\vert $可以有

$$\begin{align}
\vert x\vert = & \vert (x+x_1 +\cdots +x_n ) -(x_1 +\cdots +x_n )\vert \\
\leq & \vert x+x_1 +\cdots +x_n \vert -\vert x_1 +\cdots +x_n \vert \\
\leq & \vert x+x_1 +\cdots +x_n \vert -\vert x_1 \vert +\cdots +\vert x_n \vert ,
\end{align} $$

移项即可.

解不等式:

$22$.$\vert x+1\vert < 0.01.$

解:由$\vert x+1\vert < 0.01$推得$-0.01 < x+1 < 0.01 $,所以,$-1.01 < x < -0.99$.

$23$.$\vert x-2 \vert \geq 10$.

解:由$\vert x-2 \vert \geq 10$推得$x-2\geq 10$或$x-2\leq -10$,所以,$x\geq 12$或$x\leq -8$.

$24$.$\vert x\vert > \vert x+1\vert $.

解:两边平方,即得$x^2 > (x+1)^2$或$2x+1 < 0$,于是,有$x < -\dfrac{1}{2} $.

$25$.$\vert 2x-1\vert < \vert x-1\vert $.

解:两边平方,即得$(2x-1)^2 < (x-1)^2 $或$3x^2 -2x < 0$,解之,得$0 < x < \dfrac{2}{3}$.

$26$.$\vert x+2\vert +\vert x-2\vert \leq 12$.

解:利用不等式左边在绝对值号下的两个线性函数的零点为$-2$和$2$,就可以将实数全体分成为三个区间

$$(-\infty ,-2],(-2,2),[2,+\infty ).$$

然后在三个区间上分别求解不等式.这时在每个区间上都可以去掉绝对值号.

对第一个区间$(-\infty ,-2]$,不等式变成

$$-x-2+2-x=-2x\leq 12,$$

即得到$x\geq -6$.于是解为$[-6,-2]$.

同样可以在$(-2,2)$上解不等式

$$x+2+2-x=4\leq 12,$$

即当$x\in (-2,2)$时不等式总是成立的.

最后在$[2,+\infty )$上不等式变成

$$x+2+x-2=2x\leq 12,$$

这样得到解为$[2,6]$.

合并以上讨论,得到答案为$-6\leq x \leq 6$.

$27$.$\vert x+2 \vert -\vert x\vert > 1$.

解:$1+\vert x\vert < \vert x+2\vert $,将此式两端平方,化简得$2\vert x\vert < 4x+3$.再平方之,化简得$4x^2 +8x+3 >0$.于是,有

$$x > -\dfrac12 \;or \;x< -\dfrac32.$$

后者不适合,所以,$x > -\dfrac12$.

$28$.$\big| \vert x+1\vert -\vert x-1\vert \big| < 1$.

解:两端平方,化简得$x^2 +\dfrac12 < \vert x^2-1\vert $,即

$$x^2-1 > x^2+\dfrac12 \;or \;x^2-1 < -\left( x^2+\dfrac12 \right) .$$

前者不可能,所以,$x^2-1 < -\left( x^2+\dfrac12 \right) $,即$x^2 < \dfrac14$,解之得$\vert x\vert < \dfrac12$.

$29$.$\vert x(1-x)\vert < 0.05$.

解:由$\vert x-x^2 \vert < \dfrac{1}{20}$得$x^2-x+\dfrac{1}{20} > 0$或$x^2-x-\dfrac{1}{20} < 0$,解之得

$$\begin{cases} \dfrac{5-\sqrt{30}}{10} < x < \dfrac{5+\sqrt{30}}{10} \\ \dfrac{5+\sqrt{20}}{10} < x\;or \; x < \dfrac{5-\sqrt{20}}{10} ,\end{cases} $$

$$\dfrac{5-\sqrt{30}}{10} < x < \dfrac{5-\sqrt{20}}{10} \;or \; \dfrac{5+\sqrt{20}}{10} < x < \dfrac{5+\sqrt{30}}{10} .$$

$30$.证明恒等式:

$$\left( \dfrac{x+\vert x\vert }{2} \right) ^2 +\left( \dfrac{x-\vert x\vert }{2} \right) ^2 =x^2.$$

证明:$\left( \dfrac{x+\vert x\vert }{2} \right) ^2 +\left( \dfrac{x-\vert x\vert }{2} \right) ^2 =\dfrac12 x^2 +\dfrac12 x\vert x\vert +\dfrac12 x^2 -\dfrac12 x\vert x\vert =x^2.$

绝对误差和相对误差

$31$.设当测量$10\;\text{cm}$的长度时,绝对误差为$0.5\;\text{mm}$;当测量$500\;\text{km}$的距离时,绝对误差等于$200\;\text{m}$.那种测量较为精确?

解:用相对误差$\delta =\dfrac{\Delta }{\vert a\vert }$进行比较,其中$a$为测量的精确值,而$\Delta $是绝对误差.

对于前者,$\delta =\dfrac{0.5\times 0.1}{10} =0.5\%$;对于后者,$\delta =\dfrac{200}{500\times 1000} =0.04\%$.

所以,后者测量较为精确.

$32$.设数$x=2.3752$的相对误差为$1\%$.试确定此数包含多少位精确数字?

解:因为$\dfrac{\Delta }{2.3752} =0.01$,所以$\Delta =0.023752 $.

因而,此数包含两位精确数字.

$33$.设数$x=12.125$包含$3$位精确数字,试求此数的相对误差.

解:因为$x$包含三位精确数字,所以$\Delta < 0.05$.于是得相对误差

$$\delta =\dfrac{\Delta }{\vert x\vert } < \dfrac{0.05}{12.125} < 0.42\%$$

即$\delta < 0.42\%$.

$34$.一个矩形的边长等于:

$$x=2.50\;\text{cm} \pm 0.01\;\text{cm} ,\quad y=4.00\;\text{cm} \pm 0.02\;\text{cm} .$$

此矩形的面积$S$介于什么范围内?当其边长取平均值时,矩形面积的绝对误差$\Delta $和相对误差$\delta $是多少?

解:$S_{\min } =(2.50-0.01)(4.00-0.02)=9.9102(\text{cm} ^2)$,

$S_{\max }=(2.50+0.01)(4.00+0.02)=10.0902(\text{cm} ^2)$,

$S_{\min} \leq S\leq S_{\max } $,$S_{平均} =2.50\times 4.00 =10(\text{cm}^2)$,

$\Delta _1 =10.0902-10 =0.0902(\text{cm} ^2) $,$\Delta _2=10-9.9102=0.0898(\text{cm}^2)$,

$\Delta \leq \max{(\Delta _1 ,\Delta _2 )}$,$\delta =\dfrac{\Delta }{10} \leq \dfrac{0.0902}{10} < 0.91\%$.

$35$.一个物体的质量$m=12.59\;\text{g} \pm 0.01\;\text{g}$,其体积$V=3.2\;\text{cm}^3 \pm 0.2\;\text{cm}^3 $.若对物体的质量和体积都取其平均值,试求物体的密度,并估计密度的绝对误差和相对误差.

解:密度$C=\dfrac{12.59}{3.2} \;\text{g/cm}^3 =3.93\;\text{g/cm}^3$.

$C_{\max} =\dfrac{12.60}{3.0}\;\text{g/cm}^3 =4.20\;\text{g/cm}^3$,$C_{\min} =\dfrac{12.58}{3.4}\;\text{g/cm}^3 =3.70\;\text{g/cm}^3$,$C_{\min} \leq C\leq C_{\max }$,

$\Delta _1 =C_{\max} -C=0.27\;\text{g/cm} ^3 $,$\Delta _2=C-C_{\min} =0.23\;\text{g/cm}^3$;$\Delta \leq \max{(\Delta _1 ,\Delta _2 ) } =0.27\;\text{g/cm} ^3$;

一般地,密度为$(3.93\pm 0.27)\;\text{g/cm} ^3 $;$\delta \leq \dfrac{0.27}{3.70} < 7.3\%$.

$36$.设圆半径$r=7.2\;\text{m} \pm 0.1\;\text{m} $.若取$\pi =3.14$,则求出的圆面积的最小相对误差是多少?

解:圆面积$A=\pi \times 7.2^2 \approx 51.84\pi(\text{m}^2)$

$$\Delta _1 =\pi (7.2+0.1)^2 -\pi \cdot 7.2^2 =1.45\pi .$$

$$\Delta _2 =\vert \pi(7.2-0.1)^2 -\pi \cdot 7.2^2 \vert =1.43\pi .$$

$$\Delta \leq \max{(\Delta _1 ,\Delta _2 )} =1.45\pi (\text{m}^2 )$$

即一般的圆面积$A$为$(51.84\pm 1.45)\pi (\text{m} ^2)$,故

$$\delta \leq \dfrac{1.45\pi }{50.41\pi} < 2.88\% .$$

$37$.已测得长方体各边长为

$$x=24.7\;\text{m} \pm 0.2\;\text{m} ,\quad y=6.5\;\text{m} \pm 0.1\;\text{m} ,\quad z=1.2\;\text{m} \pm 0.1\;\text{m} .$$

此长方体的体积$V$介于什么范围内?若各测量结果都取其平均值,则能够以什么样的绝对误差和相对误差来计算该长方形的体积?

解:$24.5\times 6.4\times 1.1\leq V\leq 24.9\times 6.6\times 1.3$,即$172.480\;\text{m}^3 \leq V \leq 213.642\;\text{m}^3 $.

当$x,y,z$均取平均值时,

$$V=24.7\times 6.5\times 1.2 =192.660(\text{m}^3).$$

$$\Delta _1 =213.642-192.660=20.982(\text{m}^3),$$

$$\Delta _2 =192.660-172.480=20.180(\text{m}^3).$$

于是,$\Delta \leq 20.982(\text{m}^3)$;$\delta \leq \dfrac{20.982}{172.480} \approx 12.2\%$.

$38$.设正方形的边长$x$满足$2\;\text{m} < x < 3\;\text{m}$.为了计算此正方形的面积时能够精确到$0.001\;\text{m}^2$,应当以什么样的误差来测量边长?

解:按题设我们有$0 < x^2-4 < 0.001$或$0 < 9-x^2 < 0.001 $,解之得

$$2.99983 < x < 3\;or \; 2 < x < 2.00024.$$

因此,$\Delta $取二者中误差较小者,即

$$\Delta \leq 0.00017(\text{m})=0.17(\text{mm}),$$

故当边长$x$的绝对误差不超过$0.17\;\text{mm}$时,就能使此正方形的面积精确到$0.001\;\text{m}^2$.

$39$.设矩形每边的长皆不超过$10\;\text{m}$.为了在计算其面积时精确到$0.01\;\text{m}^2$,以什么样的绝对误差$\Delta $来测量矩形的边$x$与$y$才是足够的?

解:按题设我们有$(x+\Delta )(y+\Delta )-xy\leq 0.01$,即$\Delta ^2 +(x+y)\Delta \leq 0.01$,由于$x\leq 10$及$y\leq 10$,所以只要$\Delta ^2 +20\Delta \leq 0.01$或$\Delta ^2 +20\Delta -0.01 \leq 0$即可.解之,得

$$\Delta \leq \dfrac{-20+\sqrt{20^2+0.04}}{2} =-10+\dfrac{20.00099}{2} =0.000499 < 0.0005(\text{m}).$$

$40$.设$\delta (x)$及$\delta (y)$为数$x$和$y$的相对误差,$\delta (xy)$为数$xy$的相对误差.求证:

$$\delta (xy)\leq \delta (x) +\delta (y) +\delta (x)\delta (y) .$$

证明:设$x=a+\Delta _x ,y=b+\Delta _y $,其中$a$及$b$分别是$x$及$y$的精确值,$\Delta _x $及$\Delta _y$是绝对误差,则有

$$xy-ab=b\Delta _x +a\Delta _y +\Delta _x \cdot \Delta _y .$$

于是,

$$\Delta =\vert xy-ab\vert \leq \vert b\vert \cdot \Delta _x +\vert a\vert \cdot \Delta _y +\Delta _x \cdot \Delta _y .$$

最后得

$$\delta (xy) =\dfrac{\Delta }{\vert ab\vert } \leq \dfrac{\Delta _x}{\vert a\vert } +\dfrac{\Delta _y }{\vert b\vert } +\dfrac{\Delta _x }{\vert a\vert } \cdot \dfrac{\Delta _y }{\vert b\vert } ,$$

此即

$$\delta (xy) \leq \delta (x) +\delta (y) +\delta (x) \delta (y) .$$

文章目錄
  1. 1. 数学归纳法
  2. 2. 有理数集的分割
  3. 3. 确界的定义与性质
  4. 4. 含有绝对值的不等式
  5. 5. 绝对误差和相对误差